Last visit was: 23 May 2024, 07:34 It is currently 23 May 2024, 07:34
Close
GMAT Club Daily Prep
Thank you for using the timer - this advanced tool can estimate your performance and suggest more practice questions. We have subscribed you to Daily Prep Questions via email.

Customized
for You

we will pick new questions that match your level based on your Timer History

Track
Your Progress

every week, we’ll send you an estimated GMAT score based on your performance

Practice
Pays

we will pick new questions that match your level based on your Timer History
Not interested in getting valuable practice questions and articles delivered to your email? No problem, unsubscribe here.
Close
Request Expert Reply
Confirm Cancel
SORT BY:
Date
Tags:
Show Tags
Hide Tags
Intern
Intern
Joined: 11 Jul 2013
Posts: 27
Own Kudos [?]: 6 [0]
Given Kudos: 35
Send PM
Current Student
Joined: 18 Aug 2014
Posts: 303
Own Kudos [?]: 271 [0]
Given Kudos: 80
Send PM
Manager
Manager
Joined: 22 Feb 2016
Posts: 67
Own Kudos [?]: 52 [0]
Given Kudos: 208
Location: India
Concentration: Economics, Healthcare
GMAT 1: 690 Q42 V47
GMAT 2: 710 Q47 V39
GPA: 3.57
Send PM
Intern
Intern
Joined: 01 Sep 2016
Posts: 34
Own Kudos [?]: 39 [0]
Given Kudos: 93
Send PM
Re: Each bank in the town of La Rinconada has only a single set of locking [#permalink]
Thank you again for this amazing explanation.I fell for C


daagh wrote:
The point here is that the conclusion says that the double- door system is a security measure. We need to weaken this thinking. So, any choice that augments the security aspect is irrelevant. We need to look a totally different reason other than security against robbery or any such things. Therefore, B simply weighs in. We can ignore C, because it strengthens the argument that extra security measures are required in the second town
B is the correct choice
Intern
Intern
Joined: 08 Jul 2017
Posts: 2
Own Kudos [?]: 0 [0]
Given Kudos: 6
Send PM
Re: Each bank in the town of La Rinconada has only a single set of locking [#permalink]
ravstime wrote:
B clearly is the winner.

It provides different cause than that mentioned in the Premise. The author while arriving at a conclusion believe that only 1 cause can lead to the effect. If different cause is having the same intended effect, author's conclusion will gets weakened.


Hu ravstime,
" If a different cause is having the same intended effect, author's conclusion will get weakened." is the thumb rule for getting answer choice for Weaken questions. However, you can't fully justify the reason B is correct. It is very far fetched to bring the temperature aspect in this passage. What if I claim that Option C is valid because controlling the doors is the new technology the country has brought upon trial basis and has no relevance with the robbery. As mentioned earlier somewhere in the page, the bank can have pre-installed temperature controlled equipment which will nullify the temperature difference.
Manager
Manager
Joined: 05 Nov 2014
Posts: 79
Own Kudos [?]: 83 [0]
Given Kudos: 113
Location: India
Concentration: Strategy, Operations
GMAT 1: 580 Q49 V21
GPA: 3.75
Send PM
Re: Each bank in the town of La Rinconada has only a single set of locking [#permalink]
Each bank in the town of La Rinconada has only a single set of locking doors at its entrance. In the town of Inverness, on the other hand, the entrances to nearly all banks are equipped with two sets of locking doors, operated by a mechanism that allows only one set of doors to be open at a time. It is clear, then, that banks in Inverness experience more robbery attempts than do those in La Rinconada, and have thus adopted the extra doors as a security measure.

Which of the following, if true, most weakens the argument above?

a) Last year the number of bank robberies in La Rinconada was almost one-half greater than the corresponding figure for the previous year.
b) Inverness is known for its harsh winters, while the climate of La Rinconada is quite temperate year-round.
c) The mechanism of the double doors used by banks in Inverness allows bank security personnel to lock the doors remotely.
d) Bank robbery attempts are typically unsuccessful, and, even when the robbers do manage to escape with stolen money, the sum is usually quite small.
e) Inverness has almost twice as many police officers per capita as does La Rinconada.

Error Analysis:

In a weaken question, we may either weaken the assumption directly or provide an alternate reason.
In this argument, the comparison is made between the type of doors provided at La Rinconada and Inverness. The reason provided for the double door is the security purpose.
If we are able to provide an alternate reason or weaken the assumption made by the author, we are sorted.!!!

Option A- Irrelevant as it compares robberies at La Rinconada during last year and previous year.
Option B. Correct, as it provides an alternative reason.
Option C- Strengthens the argument. Opposite.
Option D- Irrelevant again.
Option E-Irrelevant. Doesn't address the issue.

Therefore the answer is Option B.
Manager
Manager
Joined: 05 Nov 2014
Posts: 79
Own Kudos [?]: 83 [0]
Given Kudos: 113
Location: India
Concentration: Strategy, Operations
GMAT 1: 580 Q49 V21
GPA: 3.75
Send PM
Re: Each bank in the town of La Rinconada has only a single set of locking [#permalink]
Rosicky wrote:
ravstime wrote:
B clearly is the winner.

It provides different cause than that mentioned in the Premise. The author while arriving at a conclusion believe that only 1 cause can lead to the effect. If different cause is having the same intended effect, author's conclusion will gets weakened.


Hu ravstime,
" If a different cause is having the same intended effect, author's conclusion will get weakened." is the thumb rule for getting answer choice for Weaken questions. However, you can't fully justify the reason B is correct. It is very far fetched to bring the temperature aspect in this passage. What if I claim that Option C is valid because controlling the doors is the new technology the country has brought upon trial basis and has no relevance with the robbery. As mentioned earlier somewhere in the page, the bank can have pre-installed temperature controlled equipment which will nullify the temperature difference.



Option C is strengthening and not weakening.
Intern
Intern
Joined: 22 May 2014
Status:Manager to Damager!
Affiliations: MBA
Posts: 46
Own Kudos [?]: 20 [0]
Given Kudos: 286
Location: United States
Send PM
Re: Each bank in the town of La Rinconada has only a single set of locking [#permalink]
I think it is a bad question..

Only the one who has experienced/lived in cold countries can really grasp the meaning of the option B.
Someone who is living in warm country can never understand the logic behind two doors! He/She will immediately eliminate the option B!

And such questions donot appear in official GMAT !
Current Student
Joined: 14 Nov 2016
Posts: 1173
Own Kudos [?]: 20811 [1]
Given Kudos: 926
Location: Malaysia
Concentration: General Management, Strategy
GMAT 1: 750 Q51 V40 (Online)
GPA: 3.53
Send PM
Re: Each bank in the town of La Rinconada has only a single set of locking [#permalink]
1
Kudos
voodoochild wrote:
Each bank in the town of La Rinconada has only a single set of locking doors at its entrance. In the town of Inverness, on the other hand, the entrances to nearly all banks are equipped with two sets of locking doors, operated by a mechanism that allows only one set of doors to be open at a time. It is clear, then, that banks in Inverness experience more robbery attempts than do those in La Rinconada, and have thus adopted the extra doors as a security measure.

Which of the following, if true, most weakens the argument above?

(A) Last year the number of bank robberies in La Rinconada was almost one-half greater than the corresponding figure for the previous year.

(B) Inverness is known for its harsh winters, while the climate of La Rinconada is quite temperate year-round.

(C) The mechanism of the double doors used by banks in Inverness allows bank security personnel to lock the doors remotely.

(D) Bank robbery attempts are typically unsuccessful, and, even when the robbers do manage to escape with stolen money, the sum is usually quite small.

(E) Inverness has almost twice as many police officers per capita as does La Rinconada.


OFFICIAL EXPLANATION


(1) Identify the Question Type
The question asks us to weaken the argument presented, so this is a Weaken the Conclusion question.

(2) Deconstruct the Argument
The argument attempts to explain why almost all banks in Inverness have separately locking double doors, in contrast to the total lack of such doors in the town of La Rinconada. The argument concludes that the double doors must be an extra security measure against crime, and that, by extension, the rate of the relevant crime (here, bank robbery) must be higher in Inverness than in La Rinconada.

(3) State the Goal
For Weaken questions, we have to find an answer that makes the conclusion at least a little less likely to be true or valid.

The explanation presented in the argument does seem reasonable - certainly, one reason for using double doors could be security concerns. The argument can be attacked, though, on the grounds that it claims this must be the reason. If there are other possible reasons for using the double doors, then the claim that the doors are used due to security concerns and a higher robbery rate becomes less likely to be true.

(4) Work from Wrong to Right

(A) The year-over-year growth rate of bank robberies in La Rinconada alone is not helpful in making a comparison between La Rinconada and another town.

(B) CORRECT. The door mechanism in Inverness creates an effective barrier against harsh winter winds and cold air: At no point will both sets of doors be open at the same time, so the introduction of cold air into the bank will be minimized relative to a typical single door opening. This choice thus provides an alternative rationale for the presence of the double doors, weakening the force of the original conclusion.

(C) This choice actually strengthens the argument: If the double doors are equipped with such a feature, it becomes more reasonable to speculate that they were installed as a safeguard against crime. Also, this choice can be eliminated immediately on the grounds that it doesn't distinguish at all between the two towns, and thus provides no reason for the door system to be present in Inverness but not also in La Rinconada.

(D) This choice does not address the argument's conclusion: the reason for the presence of double doors in Inverness banks and the lack of double doors in La Rinconada banks.

(E) The proportion of citizens who are police officers does not provide meaningful information about bank-robbery rates in the two towns, or about the reason for the presence of double doors in Inverness banks and the lack of double doors in La Rinconada banks.
Manager
Manager
Joined: 30 Aug 2017
Posts: 71
Own Kudos [?]: 18 [0]
Given Kudos: 250
Location: Korea, Republic of
GMAT 1: 700 Q51 V31
GPA: 3.68
Send PM
Re: Each bank in the town of La Rinconada has only a single set of locking [#permalink]
How can I find relationship between two door and weather?

Why only me think it as a problem.


I feel frustrated
Manager
Manager
Joined: 15 Aug 2016
Posts: 168
Own Kudos [?]: 56 [0]
Given Kudos: 156
Location: India
Concentration: Technology, Operations
GMAT 1: 690 Q49 V35
GPA: 3.84
WE:Operations (Consulting)
Send PM
Re: Each bank in the town of La Rinconada has only a single set of locking [#permalink]
Really Strange OA.
How would we know that double doors are used as thermostats? Or are temperature regulators or help keep the inside room warm?
C maybe the closest.
At least in C we can assume that the double doors system is not used for security but for convenience.Controlling it remotely by no way means that they would be much secure. What if the lock system is compromised for the sake of convenience?
Re: Each bank in the town of La Rinconada has only a single set of locking [#permalink]
Hi Expert, is it necessary to have 2 consecutive doors in Inverness if it has harsh winters? Thanks__
Manager
Manager
Joined: 01 Nov 2018
Posts: 69
Own Kudos [?]: 32 [0]
Given Kudos: 8
GMAT 1: 690 Q48 V35
GPA: 3.88
Send PM
Re: Each bank in the town of La Rinconada has only a single set of locking [#permalink]
You need to have outside knowledge to get this one. I eliminated all the answer choices and didn't know about the relationship between temperature and doors. Rather unfair question but the answer makes sense.
Senior Manager
Senior Manager
Joined: 17 Aug 2018
Posts: 349
Own Kudos [?]: 313 [0]
Given Kudos: 254
Location: United States
WE:General Management (Other)
Send PM
Re: Each bank in the town of La Rinconada has only a single set of locking [#permalink]
Dear experts,

Would answer choice A be correct if it were re-phrased as follows?

(A) Last year the number of bank robberies in La Rinconada was almost one-half greater than the corresponding figure in Inverness.

If that was the answer option, it would show that the number of robberies in Inverness is smaller compared to that in La Rinconada, so there must be some "other" reason why banks in Inverness have double doors. Is my reasoning right?

Thank you.
Target Test Prep Representative
Joined: 24 Nov 2014
Status:Chief Curriculum and Content Architect
Affiliations: Target Test Prep
Posts: 3480
Own Kudos [?]: 5156 [1]
Given Kudos: 1431
GMAT 1: 800 Q51 V51
Send PM
Re: Each bank in the town of La Rinconada has only a single set of locking [#permalink]
1
Kudos
Expert Reply
mykrasovski wrote:
Dear experts,

Would answer choice A be correct if it were re-phrased as follows?

(A) Last year the number of bank robberies in La Rinconada was almost one-half greater than the corresponding figure in Inverness.

If that was the answer option, it would show that the number of robberies in Inverness is smaller compared to that in La Rinconada, so there must be some "other" reason why banks in Inverness have double doors. Is my reasoning right?

Thank you.

Your suggested version of choice (A) basically directly contradicts a conclusion presented by the argument. While clearly a statement that directly contradicts a conclusion presented by the author of the argument would call into question the validity of the conclusion of the argument, such a choice is not likely to ever appear in a Critical Reasoning question.

Having said that, this question is a perhaps a bit flawed, because its argument contains two conclusions: (1) that banks in Inverness experience more robbery attempts than do those in La Rinconada and (2) that banks in Inverness have thus adopted the extra doors as a security measure. The arguments presented in Weaken questions typically contain one conclusion, and our task is to weaken the support for or call into question the validity of that one conclusion.

So, it's understandable that you would wonder whether a choice that would directly contradict the first of those two conclusions would weaken the argument. All the same, the GMAT would not use such a choice.
Senior Manager
Senior Manager
Joined: 17 Aug 2018
Posts: 349
Own Kudos [?]: 313 [0]
Given Kudos: 254
Location: United States
WE:General Management (Other)
Send PM
Re: Each bank in the town of La Rinconada has only a single set of locking [#permalink]
MartyTargetTestPrep thanks for your feedback. While it is good to know that such questions are unlikely to appear at the GMAT, I recall seeing similar questions here at the gmatclub. Unfortunately, I do not remember the source(s).

Basically, the lesson learned from this question is that whenever the argument makes a bold conclusion and the goal is to weaken the argument, one should attempt to disprove the reasoning & conclusion by searching for an alternative explanation. For instance, another "good" weakener could be a statement that says that "Banks in Inverness were built by a contractor that only installs double-doors".


Would you agree?
Target Test Prep Representative
Joined: 24 Nov 2014
Status:Chief Curriculum and Content Architect
Affiliations: Target Test Prep
Posts: 3480
Own Kudos [?]: 5156 [0]
Given Kudos: 1431
GMAT 1: 800 Q51 V51
Send PM
Re: Each bank in the town of La Rinconada has only a single set of locking [#permalink]
Expert Reply
mykrasovski wrote:
MartyTargetTestPrep thanks for your feedback. While it is good to know that such questions are unlikely to appear at the GMAT, I recall seeing similar questions here at the gmatclub. Unfortunately, I do not remember the source(s).

Basically, the lesson learned from this question is that whenever the argument makes a bold conclusion and the goal is to weaken the argument, one should attempt to disprove the reasoning & conclusion by searching for an alternative explanation. For instance, another "good" weakener could be a statement that says that "Banks in Inverness were built by a contractor that only installs double-doors".

Would you agree?

I would agree that your example involving the contractor is a great weakener for the argument that appears in this question and would be a solid correct answer to this question.

At the same time, I disagree with the following statement:

Quote:
whenever the argument makes a bold conclusion and the goal is to weaken the argument, one should attempt to disprove the reasoning & conclusion by searching for an alternative explanation.

While finding an alternative explanation for an effect is a great way to call into question the validity of a cause-and-effect conclusion such as the one that appears in this question, there are a variety of ways to weaken arguments, and there is more than one way to call into question a cause-and-effect conclusion.

So, while, yes, when you have to call into question a cause-and-effect conclusion, you may indeed find that the correct answer presents an alternative cause for the observed effect, it could be that the correct answer works in another way, such as by showing that when the supposed cause is not present the observed effect still is.

So, you have to keep an open mind, and you would probably benefit from learning about other ways in which arguments are typically weakened.
Senior Manager
Senior Manager
Joined: 17 Aug 2018
Posts: 349
Own Kudos [?]: 313 [1]
Given Kudos: 254
Location: United States
WE:General Management (Other)
Send PM
Each bank in the town of La Rinconada has only a single set of locking [#permalink]
MartyTargetTestPrep thanks for elaborating on the matter. Indeed, there are other ways to weaken / strengthen an argument that relies on cause-effect relationship. PowerScore CR Bible presents five ways how that can be done, and the alternative cause to the effect is just one of the five methods.

Thanks for a good discussion!
Re: Each bank in the town of La Rinconada has only a single set of locking [#permalink]
MartyTargetTestPrep wrote:
mykrasovski wrote:
MartyTargetTestPrep thanks for your feedback. While it is good to know that such questions are unlikely to appear at the GMAT, I recall seeing similar questions here at the gmatclub. Unfortunately, I do not remember the source(s).

Basically, the lesson learned from this question is that whenever the argument makes a bold conclusion and the goal is to weaken the argument, one should attempt to disprove the reasoning & conclusion by searching for an alternative explanation. For instance, another "good" weakener could be a statement that says that "Banks in Inverness were built by a contractor that only installs double-doors".

Would you agree?

I would agree that your example involving the contractor is a great weakener for the argument that appears in this question and would be a solid correct answer to this question.

MartyTargetTestPrep
Sir, could you help me by explaining how the highlighted part weakens the argument?
Thanks__
Target Test Prep Representative
Joined: 24 Nov 2014
Status:Chief Curriculum and Content Architect
Affiliations: Target Test Prep
Posts: 3480
Own Kudos [?]: 5156 [0]
Given Kudos: 1431
GMAT 1: 800 Q51 V51
Send PM
Re: Each bank in the town of La Rinconada has only a single set of locking [#permalink]
Expert Reply
Asad wrote:
MartyTargetTestPrep wrote:
mykrasovski wrote:
MartyTargetTestPrep thanks for your feedback. While it is good to know that such questions are unlikely to appear at the GMAT, I recall seeing similar questions here at the gmatclub. Unfortunately, I do not remember the source(s).

Basically, the lesson learned from this question is that whenever the argument makes a bold conclusion and the goal is to weaken the argument, one should attempt to disprove the reasoning & conclusion by searching for an alternative explanation. For instance, another "good" weakener could be a statement that says that "Banks in Inverness were built by a contractor that only installs double-doors".

Would you agree?

I would agree that your example involving the contractor is a great weakener for the argument that appears in this question and would be a solid correct answer to this question.

MartyTargetTestPrep
Sir, could you help me by explaining how the highlighted part weakens the argument?
Thanks__

The author of the argument arrives at a cause-and-effect conclusion - that the cause of the use of double doors in Inverness is robbery attempts.

Information that the banks in Inverness were built by a contractor who installs only double doors provides an alternative possible cause of the use of double doors in Inverness - double doors were the only option that the banks had given the contractor whom they were using.

Since it provides an alternative possible reason for the presence of the double doors, the information that the banks in Inverness were built by a contractor who uses only double doors serves to call into question the conclusion that the use of double doors in Inverness is the result of robbery attempts.
GMAT Club Bot
Re: Each bank in the town of La Rinconada has only a single set of locking [#permalink]
   1   2   3   
Moderators:
GMAT Club Verbal Expert
6936 posts
GMAT Club Verbal Expert
238 posts
CR Forum Moderator
832 posts